Management''s financial statement assertions

Assignment Help Econometrics
Reference no: EM131046778

(C) The requirement is to give the financial statement assertion that is most likely to be the focus of an auditor in his procedures when considering accounts payable. The simple net profit equation is revenue minus expense equals profit. Expenses are created when accounts payable are booked. If management wants to keep profit high they can either report higher sales or lower expense. Choice (C), completeness, is the one assertion that applies most to accounts payable because if accounts payable owed are omitted, net income is too high (i.e. overstated). Choice (A) is false because the existence assertion deals with whether recorded accounts payable are overstated, understating net income, something management is unlikely to let happen. Choice (B) is mistaken because payables have few onerous disclosures. Choice (D) is not the case because there is usually no valuation questions regarding payables - they are valued at the cost of the related acquisition unless there is a chance of non-payment or reduced payments. See ISA 500 for more information on management's financial statement assertions.

Reference no: EM131046778

Questions Cloud

Describe five audit procedures that you believe : Describe what you believe will be the critical and significant auditing concerns in Borowski's General Audit Plan.
Hypotheses to test the police department claim : An urban police department claims that the proportion of crimes committed after dusk will fall below the current level of 0.84 if LED streetlights are installed. Specify the null and alternative hypotheses to test the police department's claim.
Opportunity costs and benefits of the decision : Describe an investment decision your company has made. Compute the opportunity costs and benefits of the decision. Did your company make the right decision? If not, what would you do differently? Compute the NPV of the investment.
Probability that the first student chosen : The teacher is going to choose two of the students at random. What is the probability that the first student chosen will be a girl and the second will be a boy? Write your answer as a fraction in simplest form.
Management''s financial statement assertions : Choice (B) is mistaken because payables have few onerous disclosures. Choice (D) is not the case because there is usually no valuation questions regarding payables - they are valued at the cost of the related acquisition unless there is a chance o..
What is the most likely alternate procedure auditors : Choice (C) is mistaken because the lack of a reply does not necessarily mean the account is misstated.
Draw a histogram of emxp : Draw a histogram of EMXP. Does EMXP have an approximately normal distribution? Does it have an obvious skew (left or right) and any obvious outliers? If an analysis requires that EMXP have a normal distribution, do you think that your results will..
Proper inventory pricing pertain to the valuation : are incorrect because proper accounting requires inventory be properly valued (valuation assertion) and damaged and obsolete items and proper inventory pricing pertain to the valuation.
Fraction in simplest form : There are 7 people fishing at Lake Connor: 5 have fishing licenses, and 2 do not. An inspector chooses two of the people at random. What is the probability that the first person chosen has a license and the second one does not? Write your answer a..

Reviews

Write a Review

 

Econometrics Questions & Answers

  Find the four-firm concentration ratios for the industries

Find the four-firm concentration ratios for the following industries: fluid milk (311511), women's and girl's cut & sew dresses (315233), envelopes (322232), electronic computers (334111).

  Concentration rate and the herfindahl-hirschman index

If a corporation operates in a highly competitive industry and competes against many other companies. In the last some years, many new companies have entered the industry and firm now earns a return on investment very close to prevailing interest rat..

  What is the perfectly competitive equilibrium

Demand P (q)=100-2q Total Costs C(q)=10+20q . Calculate marginal cost for a firm in this industry. The marginal cost is graphed below, what is special or interesting about this marginal cost function. Verify the monopolist equilibrium is (Q=20,P=6..

  Determine which lease is the preferred one

As an alternative, the company offers a 24-month lease with a single up-font payment of $ 12,780 plus a $500 refundable security deposit. The security deposit will be refunded at the end of 24-month lease.

  What is the atc per poster but not per thousand

The market price is currently $1 per poster. She has fixed cost of $250. Her variable cost are $1000 for the first thousand posters, $800 for te second thousand, and then $750 for each additionl thousand posters.

  An apple farmer can only produce in quantities of 1 100 200

an apple farmer can only produce in quantities of 1 100 200 300 400 or 500 bushels and has the following costsbushels

  What is total number of workers that will be added to city a

Newton exports all their products outside the city and Garfield only sells their products within the city. Newton experiences growth, so it hires 100 more workers, all of whom come to reside in city A. Because of this, Garfield - at the first stag..

  Where will the price settle in the long run

1. How do you know that the firm represented in the graph above is a purely competitive firm 2. To maximize profits, this firm will produce at what output level (one letter) 3. Explain why this MR=MC position is the profit-maximizing position for any..

  Which economy did real gdp per person grow

it also gives each economy's average annual growth rate over this period. for example, real GDP per persion in senegal was $1,776 in 1960 and actually declined to $1,571 by 2000. Senegal's average annual growth rate during this period was -0.31%

  How much must invest in year to earn

In order to have money available for replacing their family vehicle, a couple planned to have $220,000 available in 10 years by investing. If they plan to increase their savings by 10% each year, how much must they invest in year 1 if they expect ..

  Find an expression for the breakeven price

Suppose a firm operates as a price taker in a perfectly competitive industry. The firm's Total Cost function is given by TC = a + bQ +cQ2. Therefore the firm's marginal cost is given by b +2cQ. Find an expression for the Breakeven Price.

  What level of q maximizes net benefits

Suppose the total benefit derived from a given decision, Q, is B(Q) = 25Q - Q^2 and the corresponding total cost is C(Q) = 5 + Q^2, so that MB(Q) = 25 -2Q and MC(Q) = 2Q. What level of Q maximizes net benefits

Free Assignment Quote

Assured A++ Grade

Get guaranteed satisfaction & time on delivery in every assignment order you paid with us! We ensure premium quality solution document along with free turntin report!

All rights reserved! Copyrights ©2019-2020 ExpertsMind IT Educational Pvt Ltd